« first day (1504 days earlier)      last day (3532 days later) » 

5:01 PM
@Chris'ssis I am really bad at double sum
It looks like, I have to study harder
 
Maybe you like this more (I derived it yesterday) $$\sum_{n=1}^{\infty} (-1)^{n+1}\frac{H_{2n} H_{2n+2}}{(2n+1)(2n+2)}=$$
$$\frac{29}{64}\zeta(3)+\frac{\pi^2}{12}+\frac{5\pi^3}{192}+\log(2)+\frac{\pi}{4}\log(2)+\frac{1}{24}\log^3(2)$$
$$-G-\frac{\pi}{2}-\frac{\pi^2}{24}\log(2)-\frac{1}{4}\log^2(2)-\frac{\pi}{16}\log^2(2)-\frac14\,{_4F_3}\left(\begin{array}c\tfrac12,\tfrac12,1,1\\\tfrac32,\tfrac32,\tfrac32\end{array}\middle|\,1\right)$$
From this point I suspect I can easily derive $$\sum_{n=1}^{\infty} (-1)^{n+1}\frac{H_{2n+1} H_{2n+2}}{(2n+1)(2n+2)}$$
 
That's too difficult for me. I just want to know the easy one like I asked
 
I know all I have to do, but I'm a bit involved in another type of study.
 
Okay, good night all. XOXO
 
@Anastasiya-Romanova Wait, I misunderstood your question. I thought you wanna compute some definite integrals.
 
5:06 PM
@Chris'ssis Yes, could you help me?
 
@Anastasiya-Romanova I'll think of that.
 
No. I just want to know the series of $\dfrac{\ln(1-x)}{1+x}$ that involving harmonic number
@Chris'ssis Okay, thank you. This one too: $\dfrac{\ln(1+x)}{1-x}$. Sorry if those are too many, but if you may please help me & show me how to derive it. Thanks~
 
I just flagged my enemy's answer as not an answer, lol.
 
@WillHunting Don't be so mean. There's no enemy here
 
@Anastasiya-Romanova Sorry, I am not a saint.
 
5:20 PM
@Anastasiya-Romanova I got a double series for $$\dfrac{\ln(1+x)}{1-x}$$ if I'm not wrong.
Let me check the first one now ...
 
Uh my connection -__-
so @Chris'ssis i was saying, I'm sure one can find awesome results from this, so I wanted you to know about it :)
 
@Hippalectryon Thank you. Did you find such results? :-)
 
No, i found the article earlier today
Looking at what they do at the end with it (proving the irrationality of $\sqrt{2}$), and looking at the papers they refer to, I'm pretty sure we can dig some beautiful results
 
@Hippalectryon Sure, it might be possible.
$$\int \dfrac{\ln(1-x)}{1+x}\ dx =\sum_{k\ge1} \sum_{n\ge1} (-1)^{n+1}x^{k+n} H_k \left(\frac{x}{k+n+1}-\frac{1}{k+n}\right) $$
 
5:36 PM
@Chris'ssis Oh also i'm interested by your solution, now that i've found mine.
I still use $\lfloor n!e-1\rfloor$ though
 
@Hippalectryon I did it without pen and paper ...
 
You're you :)
I'm not that awesome
Remember I'm 16 wink wink wink
 
@Hippalectryon :D
 
hi
if you have two random variables, what's a good way of saying that you can learn a lot about one of them by looking at the other?
 
Haha, wrong answer got highest votes, haha.
Wow, this is amusing!
 
5:43 PM
@WillHunting Where ?
 
4
Q: $\gcd(a,b)$ compared to $\gcd(3a,b)$

user1227849937The question asks, what can be said about $\gcd(a,b)$ and $\gcd(3a, b)$ with $a,b\in\mathbb{Z}$. They are obviously not equal in general, as $\gcd(ax, bx)=|x|\gcd(a,b)$. One thing I tried is letting $a=n\gcd(a,b)$ and $b=k\gcd(a,b)$ so that $3a=3n\gcd(a,b)$. Then $k$ and $n$ will be coprime. I ...

 
@Anastasiya-Romanova see above the double series for the first integral. Similarly you get the representation of the second integral. Piece of cake.
 
Hello
 
Hello
 
@Hippalectryon Oh they removed the upvotes on the wrong answer.
 
5:45 PM
Hello
 
No I downvoted @WillHunting
 
I needed some help with proving that $\sum_{n=1}^{\infty}\frac{\sqrt{2n-1}\ln(4n+1)}{n(n+1)}$ converges
 
anyone understand methods behind evaluating multiple-integrals
everything i try isnt working
 
@user176201 Sure, @Chris'ssis does :)
 
I know I have to use the comparison test, but I dont know which sequence I should compare it to
 
5:47 PM
@VibhavPant What comparison test ? (what version?)
 
@Hippalectryon Limit comparison test
Where you prove $a_n \sim b_n$
 
@Anastasiya-Romanova I'm sorry. Our privacy agreement doesn't allow us to say anything. If Mhenni wishes to discuss it, that is fine.
 
$\tilde b_n$ ? @VibhavPant
@VibhavPant it's '\sim'
 
ah, thanks
@Hippalectryon so yeah, that test
 
@VibhavPant Ok so
@VibhavPant Let $u_n=\dfrac{\sqrt{2n-1}\ln(4n+1)}{n(n+1)}$
$u_n\sim?$
 
5:51 PM
$$\frac{\log(n)}{n^{3/2}}$$
 
@VibhavPant compare with $\sum\frac{\log(n)}{n^{3/2}}$ which you can do by condensation
 
Uh ?
No need
Bertrand Series
Therefore converges
 
meh. $\zeta(s)$ is analytic on $Re[s] > 1$
=P =P
 
@Chris'ssis how did you get that function
@robjohn also, what's condensation
 
@VibhavPant $\sqrt{2n-1}\sim?$
 
5:52 PM
@VibhavPant $n + 1 \sim n$, $\sqrt{2n-1} \sim n^{1/2}$
 
If I have a double integral with x going from 0 to L and y going from 0 to L, and I replace (y-x) with u, do we say u goes from -L to L?
 
$4n + 1 \sim n$
 
@BalarkaSen that requires facility with zeta and with differentiation of series
 
@VibhavPant that's just an approximation. Moreover, you series is about $$\int_1^{\infty}\frac{\log(x)}{x^{3/2}}\ dx=4$$ that is very easy to compute.
 
5:53 PM
@Chris'ssis asymptotic approximation
@robjohn I know, I was kidding.
 
@Chris'ssis integral test?
 
meh
 
Bertrand series give you the answer right away
 
@VibhavPant kind of
 
5:55 PM
@BalarkaSen not really any more than using an integral... You can do Cauchy condensation before you even hear of an integral
 
i was actually not using integrals
just little-oh approximations
 
@robjohn Cauchy is better.
 
@BalarkaSen o not oh -__-
@Chris'ssis No Bertrand :c
 
@BalarkaSen how come $4n+1 \sim n$? isnt $\lim_{n\to\infty}\frac{4n+1}{n} = 4$
 
Death to Cauchy
 
5:56 PM
@Hippalectryon lol :-)
 
@Vibhav that's the definition of asymptotics
 
@Hippalectryon isnt he already dead?
2
 
@VibhavPant $4n+1\sim4n$ not $n$
 
well, $4n + 1 \sim 4n$
put out the constant aside
 
@BalarkaSen Now, it's OK. :-)
 
5:56 PM
The constant is not to be set aside in equivalents
 
C'mon, @Chris'ssis
just a typo
=)
 
Sorry, we made a typo on your bank note :/
 
@BalarkaSen :D
 
@VibhavPant You can put the constant out of the sum
 
5:57 PM
thanks
 
@BalarkaSen you trying to show the growth rate of log vs powers is not hard but I wouldn't say it is easier than Cauchy condensation. Condensation is easy to prove and use.
 
@BalarkaSen Integral ???
 
meh meh meh
 
Mindblown
 
@Hippalectryon This image is too violent, lol
 
5:59 PM
OK, how would I prove that $\sum_{n=1}^{\infty}\frac{\ln4n}{n^{3/2}}$ converges?
 
@VibhavPant $\ln(4n)\sim\ln(n)$ btw
@VibhavPant Have you seen Bertrand series ?
 
@Hippalectryon TAKE THIS FOR BEING TYPO-NITPICKY ┻━┻ ︵ ლ(ಠ益ಠლ)
 
@BalarkaSen i'm not gonna start that again -__-
 
@VibhavPant $\log(4n) = \log(4) + \log(n) \sim \log(n)$
 
The book I am reading hasnt inroduced Bertrand series yet, @Hippalectryon
 
6:00 PM
you made a typo in the emoticon :D
 
where?
 
That was just a joke to make you rage even more -__-
Irony, people
@VibhavPant What about the Cauchy test ?
 
nah
thats later
 
@Hippalectryon (ノ゜益゜)ノ彡 (-_________-)
 
I suppose I have to use the integral test for $\frac{\ln(n)}{n^{3/2}}$
 
6:02 PM
yeah
 
Integral test ?
@Chris'ssis Give me your solution 'without pen and paper' :)
Might be different from mine
 
@Hippalectryon if $f$ is positive, decreasing, continuous, define $s_n = \sum_{k=1}^nf(k)$ and $t_n = \int^n_1f(x)dx$. Then both $\{s_n\}, \{t_n\}$ converge or diverge
 
Hm yeah that's obvious. I see.
 
@Hippalectryon $$\prod_{n=1}^N \left(\frac{t_{n+1}}{(n+1)t_n}\right)=\frac{t_{N+1}}{(N+1)!}$$ and then use the hint in the right panel.
 
@Chris'ssis That much is the easy part, I'm talking about the one after
Uh wait
Let me look at the hint
 
6:10 PM
@Hippalectryon $$\underbrace{\frac{t_n}{n!}}_{\displaystyle a_n}-\underbrace{\frac{t_{n-1}}{(n-1)!}}_{\displaystyle a_{n-1}}=\frac{1}{(n-1)!}$$
and sum up over $n=2$ to $N+1$
 
Mhm I see
Got a math exam tomorrow :c
I should say :D
 
@Hippalectryon Good luck. My spirit is with you.
 
Ghost busters, please !
 
@Hippalectryon What subject?
 
@Hippalectryon there is no real way to use KaTeX here anyway, besides the library is not as complete or debugged.
 
6:18 PM
I feel like flagging all my enemy's silly comments. But that would be too many flags @robjohn, lol. She just keeps praising this user's answers (I think hoping that he would upvote her in return...)
@robjohn What's KaTeX?
 
@WillHunting I'm not sure who your enemy is...
@WillHunting look here
 
@robjohn The one with the 5th highest rep...
@robjohn Will I get into trouble if I flag too much? There are just too many of her comments that are not constructive.
 
@WillHunting oh, I see.
 
It's time for someone to tell her to STOP her nonsense.
 
@WillHunting we deal with a lot of flags against a group of 3 or 4 and their comments to each other. I won't say who.
 
6:22 PM
@robjohn OK, I will just flag them. If a mod finds it excessive he can email me.
 
@BalarkaSen Also, isnt $\sqrt{2n-1} \sim \sqrt{2}n$
 
@WillHunting this has not gone unnoticed
 
@robjohn Like I said, it is her trick to get upvotes. I used to upvote her a lot and she upvoted me a lot, but I made it clear that we were not in collusion. Then she started to leave all her stupid comments on my post too, and I had to tell her to stop.
 
@VibhavPant why?
 
It takes chains of steel to enslave a rational human, but all the rest may be enslaved by a belief.
 
6:25 PM
@robjohn $\lim_{n\to\infty}\frac{\sqrt{2n-1}}{\sqrt{2n}} = 1$
 
@WillHunting I don't know if the comments garner upvotes, but I guess letting a group of users know that one is upvoting them a lot might.
@VibhavPant can you show that? what is the question?
 
@robjohn here what I tried $\lim_{n\to\infty}\frac{\sqrt{2n-1}}{\sqrt{2n}} = \lim_{n\to\infty}\sqrt{1-\frac{1}{\sqrt{2n}} = 1$
 
@robjohn I just flagged about 10 of her comments, lol.
 
@WillHunting someone will probably tell you to stop. I probably should have said to simply flag one with a custom message
 
@robjohn Sorry for the wrong typsetting: $\lim_{n\to\infty}\frac{\sqrt{2n-1}}{\sqrt{2n}} = \lim_{n\to\infty}\sqrt{1-\frac{1}{2n}} = 1$
 
6:31 PM
@VibhavPant well, it would be $\sqrt{1-\frac1{2n}}$, but yes. Why do you doubt that?
 
@rehband Sequences and series
 
Good luck with your test, @Hippa.
 
@Khallil Thanks :)
 
@robjohn @BalarkaSen earlier said that $\sqrt{2n-1} \sim \sqrt n$ (most likely a typo)
@robjohn which was a part of a question on proving a series in convergent
 
It was a typo
 
6:33 PM
yeah
 
@VibhavPant yeah... I was there.
 
@Hippalectryon Nice, you will crush it
 
@rehband Uh not sure
@Chris'ssis Hey I had one good problem
 
Hi, I need some help with this please : Let $X=\{f_0),f_1,f_2,\cdots\}$ where $X$ is the set of application from $\Bbb{N}$ to $\{0,1\}$. I want to show that $g$ cannot be an element of the sequence $\{f_0),f_1,f_2,\cdots\}$ where $g$ is defined : $g(n)=0$ if $f_n(n)=n$ and $g(n)=1$ if $f_n(n)=0$. Any ideas ?
 
@Chris'ssis Probably way too easy for you, but still
 
6:37 PM
@Hippalectryon I do some research now.
 
I'll give it anyway
 
@robjohn No problem. I have lots of time to respond to emails, lol.
 
@Chris'ssis Here found it
Ex 27 (ENS 2010): Let $k\geq2,(u_1,\dots,u_k)\in\mathbb{C}^k$. For $n\geq k$, let $u_{n+1}=\frac{1}k\displaystyle\sum_{j=n-k+1}^nu_j$. What is $u_n$'s limit ?
@rehband You can try too :)
 
@Hippalectryon Already doing it :D
 
7:01 PM
The solution I have uses the Gauss-Lucas theorem, so It'd be great to have an alternative
 
I'm not even familiar with that theorem :)
@Hippalectryon Is the limit k?
 
7:16 PM
@Chris'ssis What that thing you removed interesting (I didn't see)
@rehband I don't know :) I haven't written the proof on paper.However, $k$ seems a bit too simple. How did you proceed ?
 
@Hippalectryon For your question, the natural thing is to start with $k=2$ and play a bit with that case. Then you may easily extend to the other cases.
 
@Chris'ssis Can you find a method that doesn't use Gauss-Lucas ?
 
@Hippalectryon Yeah, nevermind it's wrong :P
 
@Hippalectryon I think I know the solution to your question.
 
@Chris'ssis Please share it
 
7:29 PM
@Chris'ssis What I'm interested in is the method, not the solution ;) Don't be Cleo
hehe
I can't even imagine what a book written by @Chris'ssis and @Cleo would be
 
I aspire to write a series of books covering all major branches of math. Maybe 10 volumes?
 
No, that was not OK, the starting order doesn't matter.
 
I have 750 points, 250 more to reach 1000.
 
@WillHunting I'll say good bye now in case you're gone before I come back...
 
@robjohn No need to say bye, I am not dying yet. =)
 
7:40 PM
Well, it matters
@Hippalectryon is for $k=2$ $$l=\frac{u_1+2 u_2}{3}$$?
I think the numerator behaves like $$u_1+2u_2+3u_3+\cdots +k u_k$$ for higher $k$ - but not very sure yet
 
@Chris'ssis This seems to agree with my observations so far
 
No, wait a second
 
@Chris'ssis Do you already have a proof for the case $k=2$ ?
 
@Hippalectryon $$l= \frac{u_1+2u_2+3u_3+\cdots +k u_k}{3(k-1)}$$
@rehband A second, I'm still wrong ... (not far from the truth)
 
@Chris'ssis =)
 
7:55 PM
@robjohn How can we prove that if $2^x, 3^x, 5^x, 7^x ...$ are all integer then $x$ is an integer too?
 
@Hippalectryon Final answer
$$l= \frac{u_1+2u_2+3u_3+\cdots +k u_k}{\displaystyle \binom{k+1}{2}}$$ to put it nicely
 
@Chris'ssis Sounds like you're answering a question on Who Wants to be a Millionaire
 
lol
 
@rehband hahaha
 
and she probably has become a millionaire too
@robjohn You here? :)
 
7:59 PM
@Sawarnik We shall find out after a short advertisement break
 
:D
How much time will the break last!
 

« first day (1504 days earlier)      last day (3532 days later) »